« first day (36 days earlier)      last day (538 days later) » 
00:00 - 16:0016:00 - 00:00

4:02 PM
0
Q: Finding the summation of equation...

Mathingwhat will the summation be for the following equation: 45((n(n+1))/2)

0
Q: A short question about trig identity of sum of angles

ILoveChessAre those expressions $ 4\sin(x)(\sin(x)\cos(2x) + \cos(x)\sin(2x) $ and $4\sin(x)\sin(3x)$ equivalent?

Words such as question are uninformative in titles. Please edit the title so that it better describes the specifics of your question. Do not hesitate to make it longer or include a formula if needed. More tips here. (autocomment)Normal Human 21 secs ago
0
Q: Range of $tan(A)tan(B)$

Sanchayan Dutta In a triangle if $\tan A<0$ then values of $\tan (B)\tan(C)$ should be in the range $(0,1)$. (Says my book) I'm getting that $\tan (B)\tan(C)$ should be lesser than $1$ as $$tan (A)=\frac{\tan(B)+\tan(C)}{\tan(A)\tan(B) -1 }$$ But why should it be greater than $0$ ?

0
Q: Normal family of functions

user160492I have the following question and I have no idea how I can solve it. Let F be family of functions f in A(D) where D is aunit disc. if a)f(0)=2i and b) |f(z)|>1 for all z in D then Prove that F is a normal family in A(D). I tried to use montel's but this family of functions is not locally bound...

0
Q: Linear Algebra Orthogonal Projections textbook question

Newb18In my textbook it says: $\frac{4}{\sqrt{2}}$V1 + $\frac{6}{\sqrt{3}}$V2 and suddenly on the next page it says 2V1 + 2V2 is there something i'm missing here? or did they just second power the sqrt out???

Words such as question do not add information to titles. Please edit the title so that it better describes the specifics of your question. Do not hesitate to make it longer or include a formula if needed. More tips here. (autocomment)Normal Human 21 secs ago
0
Q: Assistance in finding $\lim_{n\to\infty} \frac{n-\lfloor \sqrt n \rfloor^2}{n}$

proc-self-mapsI am trying to prove $$\lim_{n\to\infty} \frac{n-\lfloor \sqrt n \rfloor^2}{n} = 0$$ given $n>0$. But I'm having difficulties dealing with the floor function. I tried splitting apart the limit like so: \begin{align*} \lim_{n\to\infty} \frac{n-\lfloor \sqrt n \rfloor^2}{n} &= \frac{\lim_{n\to\...

0
Q: Direct Limit of Grassmannians

PPRLet $X$ be a topological space and $G_n(\mathbb{C}^m)$ be the space of vector subspaces of $\mathbb{C}^m$ of codimension $n$. Let $G_n(\mathbb{C}^\infty):=\bigcup_{m=n}^{\infty}G_n(\mathbb{C}^m)$ using the natural inclusion $$G_n(\mathbb{C}^m)\hookrightarrow G_n(\mathbb{C}^{m+1})$$ Question: Is...

 
0
Q: Is it possible to get a featured tag badge?

Caleb KleveterI posted an answer in a question that had the featured tag recently that got a fair amount of up-votes, that got me wondering if you could possibly get the featured tag badge. But then I noticed that after some time the tag seems to disappear from the question. So, is it correct that it is imposs...

 
0
Q: Eigenvalue alteration counter-proof

user296905If $A$ is a square $n\times n$ matrix, with $\lambda_1,\ldots,\lambda_n$ being the eigenvalues of $A$, $v_1$ being the eigenvector associated with eigenvalue $\lambda_1$, and $d$ the column vector of dimension $n$, then can anyone provide a counterexample to show the following statement is false:...

0
Q: Continuous functions with $f^2(x)=g^2(x)$

Niklas HebeLet $f,g:[a,b]\subset \mathbf R\to \mathbf R$ be continuous functions with $f^2(x)=g^2(x)\neq 0$ for all $x\in[a,b]$. Show $f=g$ oder $f=-g$ on $[a,b]$.

Consider replacing (analysis) with a more specific tag for the relevant branch of analysis. (autocomment)Normal Human 21 secs ago
0
Q: What happens if you repeatedly take the arithmetic mean and geometric mean?

MalcolmGiven two positive real numbers, A and B, such that A<=B, take the geometric mean, giving A', and the arithmetic mean, giving B'. Repeat ad infinitum. My intuition tells me that, since both means give values between the two original numbers, they will converge as the number of repetitions approac...

Welcome to Math.SE, Malcolm. Questions tend to get more attention when they have a tag for a broad area of mathematics relevant to the question. Some of these tags might fit. (autocomment)Normal Human 21 secs ago
0
Q: Difference of random variables conditioned on their sum

SashaConsider $\Omega = [0,1] \times [0,1]$ with sigma algebra of borel sets on $[0,1]^2$. Let $P$ be the Lebesgue measure on $\Omega$. Let $$\xi(x, y) = x, \ \ \ \eta(x,y) = y.$$ How can I find $\mathbb{E}(\xi - \eta| \xi +\eta)$? I tried to find out what $\sigma( \xi + \eta)$ and I've found that $...

Questions tend to get more attention when they have a tag for a broad area of mathematics relevant to the question. Some of these tags might fit. (from a bot)Normal Human 21 secs ago
0
Q: Stuck with neice's homework!! Ostensibly a bearings question, but I need more to solve

user297011My neice asked me for help with her national 5 homework. (National 5 is a new qualification in Scotland roughly equivalent to the old standard grade, so I would have thought quite easy). I have made a crude MS paint of the problem and linked it below. http://s22.postimg.org/nyhmey2a9/nat5_maths....

0
Q: Why does the equality hold?

evindaFor $A \in \mathbb{R}^{n \times n}$ with $A=A^{T}$ we set: $$\lambda:= \max \{ \langle x, A x \rangle: ||x||_2=1\} \\ \mu:= \min \{ \langle x,A x \rangle: ||x||_2=1\}$$ Then for $x \in S_{||\cdot||_{2}}$ we have: $$\langle x, Ax \rangle \leq |\langle x, A x \rangle| \leq ||x||_2 ||Ax||_2 \leq ...

0
Q: How would I derive the closed form for this?

VetaI'm trying to calculate the variance of a martingale strategy (inital_bet = 1) with infinite bankroll. Where p = P(winning), N = number of trials to win, M = payout. Since this is a geometric variable I know N = 1/p. I deduced E[M] = 2^(1/p - 1). But I'm having trouble with var(M). I tried the...

0
Q: find a function such...

Elll exercise : A function $f$, continuous on the positive real axis, has the property that $$\int_{1}^{xy}f(t)dt =y\int_{1}^{x}f(t)dt +x\int_{1}^{y}f(t)dt$$ for all $x > 0$ and all $y > 0$. If $f (1) = 3$, compute $f (x)$ for each $x > 0$. My progress: I derive $f(xy)y = yf(x)+\int_{1}^{...

Welcome to Math.SE, Veta. This site uses MathJax formatting of formulas. More tips here. (from a bot)Normal Human 21 secs ago
0
Q: Numerical solution of the stationary Navier-Stokes equations

0xbadf00dLet $d\le 3$ and $\Omega\subseteq\mathbb R^d$ be a bounded domain. I'm considering an incompressible Newtonian fluid with uniform density $\rho_0$ and viscosity $\nu$. In this case, the stationary Navier-Stokes equations are $$\left\{\begin{matrix}(u\cdot\nabla)u&=&\displaystyle\nu\Delta u-\frac ...

Tagged pde, differential-equations. Tagged differential-equations but mentions "partial". Numerical solution of the stationary Navier-Stokes equations
0
Q: The "pepperoni pizza problem"

Ben S.This problem arose in a different context at work, but I have translated it to pizza. Suppose you have a circular pizza of radius $R$. Upon this disc, $n$ pepperoni will be distributed completely randomly. All pepperoni have the same radius $r$. A pepperoni is "free" if it does not overlap an...

Title contains problem. The "pepperoni pizza problem"
 
4:34 PM
-1
Q: Why a Zero Voted Question is Considered as a Bad One?

Darius MiliauskasA number of my question attract zero voting. I am a bit surprised that neutral attitude is considered as the bad one. Voting itself is a subjective process, for some people the question could look broad, for some too narrow, it depends on the persons knowledge at the current moment. Some question...

 
0
Q: Algebraic and geometric multiplicities (soft question)

Martín Forsberg CondeI'm aware of the formal definition of algebraic and geometric multiplicities, but I can't make much sense out of the names. What I mean is, if it were me, I would name these two quantities multiplicity one and multiplicity two, for all I know. But I'm guessing that the people who named these two...

Words such as question are uninformative in titles. Please edit the title so that it better describes the specifics of your question. Do not hesitate to make it longer or include a formula if needed. More tips here. (autocomment)Normal Human 21 secs ago
 
1
Q: Is there a license on all the code put on SE?

Paranoid PandaBy putting code on SE (by posting it in a question or answer or another form of publicly posting it on the site) is the code given some sort of license automatically if one is not declared by the owner? So what I mean is for instance if person A wrote their own program and posted it as an answer ...

 
0
Q: What is the value of $\prod_{n=2}^\infty\Big(1-\dfrac2{n(n+1)}\Big)^2$?

Saun DevIs the infinite product $\prod_{n=2}^\infty\Big(1-\dfrac2{n(n+1)}\Big)^2$ convergent ? If so , then what is it's value ? Please help . Thanks in advance

Tall formulas in titles break the layout of question lists. Please replace \dfrac with \frac in the title. (autocomment)Normal Human 21 secs ago
0
Q: Maximum value of absolute value function

mrbHow to solve the following optimization problem: $$ \max_{x_i} ~ f(x_i,x_j) = h(\max(x_i,x_j)) - x_i\qquad i=1,2 $$ where $h(\cdot)$ is a concave function. Thanks.

0
Q: Deriving probability rules

STFConsider three events $A,B,C$ and a probability measure $\mathbb{P}$. Do you know any rule linking $\mathbb{P}(A\geq B)$ with $\mathbb{P}(A\geq C)$, $\mathbb{P}(C\geq B)$? Moreover, is it true that $\mathbb{P}(A\geq B)\leq \mathbb{P}(A\geq C)+\mathbb{P}(C\geq B)$? For any rule that you sugges...

0
Q: Counting Principles problem people in line

TheUltimate AliYou have 9 boys and 6 girls in a line. In how many ways can you arrange them so that no girls stand next to each other.

Title contains problem. Short question. Counting Principles problem people in line
0
Q: Linear transformation T: M33 -> M33 defined by T(A) = 1/2(A+A^T). Determine a basis for the kernel of this mapping.

Suhail PrasathongThe linear transformation is given as T: M33 -> M33 defined by T(A) = 1/2(A+A^T). This is also known as the symmetrization operator.

0
Q: What exactly is Green's Function and why can I use it to solve harmonic oscillator problems?

bluemoonMy question is basically in the title: What exactly is Green's Function and why can I use it to solve harmonic oscillator problems? In other words, how is Green's function connected to physics problems like the harmonic oscillator? My question is motivated by a bonus question on my work...

0
Q: Notation in set thory.

k.dkhkWill someone please describe the following set in words.

 
5:11 PM
0
Q: How to politely ask the OP for feedback about your answer?

m69Sometimes you answer a question, and the OP doesn't accept or upvote it, but also doesn't downvote, or provide a comment as to why it doesn't help them. So then you're left wondering whether they used the answer and moved on, or they couldn't use the answer in their specific situation, or maybe y...

 
Short title. Short question. Question contains please. Notation in set thory.
0
Q: How to find the lower limit of this sequence?

memonto$(a_n) =(\frac{(-1)^{n+1}n}{4n-5})$ I dont understand why the lower limit isnt equal to - 1. Since it is lowest number in the set of infumums

0
Q: Limit problem with roots

Per KarlssonI'm struggling solving the following limit problem: Limit problem At first I thought I could Multiply by: conjugate But that doesn't seem to take me anywhere closer to an answer. Some help would be appreciated.

Short title. Title contains problem. Limit problem with roots
0
Q: Formula that describes the movement of a bishop in chess

Mr DI'm programming a chess game and I'm trying to validate the movements every player tries to make. Obviously, every piece can move differently and I've had no trouble validating their moves up until now. The first thing I do in order to validate the pieces is to check for collisions. In order to ...

Consider replacing (analysis) with a more specific tag for the relevant branch of analysis. (autocomment)Normal Human 20 secs ago
0
Q: Can you solve the following functional equation?

thefunkyjunkyI found the following functional equation: Find all functions $f : \Bbb R \rightarrow \Bbb R $ such that: $xf(x) + yf(y) = (x - y)f(x + y) $ for all $x, y \in \mathbb R $ Could you please help me? I think I proved that if $f(0) = 0$ then for each $x \in \Bbb Q$ $f(kx) = kf(x)$, but I don't kno...

0
Q: Where is t=0 in the domain?

Grzegorz BaltissenGiven: $f(t,u(t)) = u'(t)=t\cdot exp(u(t))$ and $u(t_0)=u_0$ We have as solution $u(t)=-ln(e^{u_0}+\frac{1}{2} t_0²-\frac{1}{2}t²)$ as here: Wolfram's solution It somehow bothers me that whatever $t_0$ is chosen, it is between the two asymptotes $\pm \sqrt{t_0²+2e^{-u_0}}$. It would mean that t...

0
Q: Higher order derivate definition

IliketoproveitI know that higher-order derivatives are defined inductively as $f^{(n)}=(f^{n-1})'$. However does this imply that $f^{(n)}(x) = \lim_{h\to0}\frac{f^{(n-1)}(x+h) - f^{(n-1)}(x)}{h}$ ?

0
Q: Orthogonal projection - creating a matrix

Anirudh GangwalI know that orthogonal projection of a vector v on a subspace W is given by proj_W(v) = (u1.v)/u1.u1 * u1 + ... + (uk.v)/uk.uk *uk where {u1,..,uk} is an orthogonal basis of W. I am suppose to find a matrix B that projects a vector on W. Now, I know that by applying the above formula with tak...

This site uses MathJax formatting of formulas. More tips here. (autocomment)Normal Human 21 secs ago
0
Q: Solve this ODE system

DreamerI have this ODE system:$$x'(t) = ax(t)+ by(t)$$ $$y'(t)= cx(t) + dy(t)$$ how do I get the coefficients? I know how to solve ODE with two variables, but now sure how to solve the system...

Short title. Short question. Solve this ODE system
0
Q: Do graphical calculators use Newton's method?

GigiSo the question is, do calculators really use Newton's method to calculate things? I mean, for some equations, is it how they get the answer?

Welcome to Math.SE, Gigi. Questions tend to get more attention when they have a tag for a broad area of mathematics relevant to the question. Some of these tags might fit. (autocomment)Normal Human 21 secs ago
0
Q: Is this conditional independence holds?

VincentI have a random variable $X$, where $0<X<1$; and a random variable $Y$. Assume $X$ and $Y$ are uncorrelated but not independent. If I let $Z=binomial(p=X)$. Is it true that $Z \perp Y|X$

Tag (independence) should not be the only tag a question has. Please add a tag for a subject area to which the question belongs. (autocomment)Normal Human 21 secs ago
0
Q: Given $A \overset{f}{\to}B$, what measures how hard it is to factor through $f$?

Eric AuldSuppose $A \overset{f}{\to} B$ is a morphism in a category (say for ease they are abelian groups). When we consider other morphisms out of $A$ (say to a fixed object $C$), suppose we ask which maps $A \overset{g}{\to} C$ factor as $A \overset{f}{\to} B \overset{\tilde{g}}{\to} C$. Here are the...

0
Q: Polynomial curve fit - where "y" is not zero

Caio Graco RodriguesI have an equation to fit a curve of phosphorus concentration Response = 1.927596E-03 + 4.631966E-04*concentration + 3.297547E-08*concentration^2 I usually use it to predict my absorbance by an know concentration solution. For example, using the concentration 11,473: f(x) = ax^2 + bx + c ; ...

Welcome to Math.SE, Caio Graco Rodrigues. This site uses MathJax formatting of formulas. More tips here. (from a bot)Normal Human 21 secs ago
0
Q: how to show that$\ Hom(\mathbb Z_n,\mathbb Q/\mathbb Z)\cong \mathbb Z_n$

parisaI want to show this but I dont have idea. $\ Hom(\mathbb Z_n,\mathbb Q/\mathbb Z)\cong \mathbb Z_n $ what the homomorphism between $\mathbb Z_n \to \mathbb Q/\mathbb Z$. thanks your help

0
Q: Could someone explain this step to me?

AngelisExuroWe consider the following setting: Let $Z_{1},...,Z_{n}$ be iid random variables with distribution function $H_{Z}$ and $u>0$ a constant. We set $M:= \sup_{n\in N} \sum_{k=1}^{n} Z_{k} $. I found the following step in a book: $P(Z_{1}\leq x_{1},...,Z_{n}\leq x_{n},M>u) = \int_{-\infty}^{x_{1}} ...

Words such as someone are uninformative in titles. Please edit the title so that it better describes the specifics of your question. Do not hesitate to make it longer or include a formula if needed. More tips here. (from a bot)Normal Human 21 secs ago
0
Q: Trapezoidal Rule

user297037enter image description here How do I solve the below equation with single application of trapezoidal rule?

Short title. Short question. Trapezoidal Rule
0
Q: Least squares optimization

clarksonIn Least Square optimization, A=\begin{pmatrix} 1 & t_1 & t_1^2 & \cdots & t_1^k \\ 1 & t_2 & t_2^2 & \cdots & t_2^k \\ \vdots & \vdots& \vdots & \ddots & \vdots \\ 1 & t_n & t_n^2 & \cdots & t_n^k \end{pmatrix} b=\begin{pmatrix} s_1 \\ s_2 \\ ...

0
Q: Can Someone approve the formula for the number of groups of order $p^2q$

PeterHere https://www2.bc.edu/~reederma/Groups.pdf on page $112$, a table of the number of groups of order $p^2q$ is given. In the explanations, there is a typo ($\frac{q+5}{5}$ instead of $\frac{q+5}{2}$) Can someone approve the following formula for $f(p,q)$, the number of groups of order $p^2...

 
6:06 PM
0
Q: Is cross-validated the appropriate place to ask advice on career-development questions in statistics?

ThomasKlauschAs far as I know there is not a stackechange site for discussing career decision questions 'in general' but it would be nice to have this for the field of methodology and statistics. Many people are looking cross-entry from other fields, especially in the context of 'big' data science. Statistics...

 
0
Q: proof of hamiltonian cycle

user297042graph Note that the graph has been constructed so that there are no cycles of length three (triangles) or four (quadrilaterals). Note also that the gra ph has a total of fifteen edges and ten vertices, and every vertex has degree 3. Now, s uppose this graph had a Hamiltonian cycle. That wo...

0
Q: How to prove at least solution to x^2+y^2=-1modp exists

SazzleHow can I show this? For every prime number, p, there is at least 1 solution (x,y) such that x^2+y^2=-1modp Thanks

0
Q: Solve $\frac{dy}{dx} + \frac{1}{x} \tan(y)= \frac{1}{x^2} \tan(y)\sin(y)$

Sanchayan Dutta How to solve the differential equation $$\frac{dy}{dx} + \frac{1}{x} \tan(y)= \frac{1}{x^2} \tan(y)\sin(y)$$ Hints please.

0
Q: The Basic Principle

Serkan KlvzIn any n+1 integers there will be a pair which differs by a multiple of n. I have tried to create a pigeon hole with numbers a0,a1,a2,...,an but i could not get a solution.

Short title. Title contains basic. Short question. The Basic Principle
0
Q: system of non-linear equations, complex substitution

fdhdI have problems solving \begin{equation} \frac{dx}{dt}=2xy; \quad \frac{dy}{dt}=1+x^2-y^2. \end{equation} I can solve similiar system easily when I have \begin{equation} \frac{dy}{dt}=2xy; \quad \frac{dx}{dt}=1+x^2-y^2, \end{equation} by introducing complex number $z=x+iy$, then $\frac{dz}{dt}=...

Tagged pde, differential-equations. system of non-linear equations, complex substitution
0
Q: Checking a calculation with mean curvature and second fundamental form

SashaLet $M$ be a 3 dimensional manifold, $N$ a surface in $M$ and $A$ the second fundamental form on $N$, $H$ the mean curvature. $h$ is the metric induced on $N$. I need to show that \begin{equation*} \int_{N_l}|A - \frac{H}{2}h|^2 = \frac{1}{2}\int_{N_l}(\lambda_1 - \lambda_2)^2. \end{equation*} w...

0
Q: A question on proper maps $f$ and embeddings into the $m$-th unit-cube

jeffreySuppose $X$ and $Y$ are topological spaces which are both embeddable as a closed subspace of $\mathbb{R}^n$ for some $n\geq 0$. Let $f\colon X\to Y$ be a proper map, i.e. a continuous map such that the pre-image of every compact set in $Y$ is compact in $X$. Do there exist a continuous map $...

0
Q: Difference Between joint probability distribution and conditional probability distribution?

user297048Can someone explain to me the difference between joint probability distribution and conditional probability distribution?

0
Q: Vector Rotation About Arbitrary Axis

aaa111I am new to this forum. I was reading this document : http://math.kennesaw.edu/~plaval/math4490/rotgen.pdf Here the author says that from this figure : http://i.imgur.com/4KyrI3e.png We can express Vper like this : T (Vper) = COS * Vper + SIN * W I dont understand this part. Can anybody exp...

Welcome to Math.SE, aaa111. This site uses MathJax formatting of formulas. More tips here. (from a bot)Normal Human 21 secs ago
0
Q: linear optimization if , then, else constraint

gülgülI want to ask how to model if x==y return 1 else return 0 function as constraints in the model?

0
Q: Uniformly most powerful test

andySuppose we are given a random sample x1, x2,..., xn from a distribution with density: f(x;θ) = θx^(θ−1) ,0 < x < 1. Find the UMP critical region of significance level α for testing H0 : θ = θ0 against H1 : θ < θ0 C={x:L(θo)/L(θ1)<Κ} ={x: θo x^(θo-1)/θ1 x^(θ1-1) Woudl this be...

This site uses MathJax formatting of formulas. More tips here. (from a bot)Normal Human 21 secs ago
0
Q: Second derivative of the Fourier tranform

user296858I have been asked to show the Fourier Transform of f''(t), however I am confused about what apporach to take. I know that there are certain properties that follow Fourier Transforms for derivatives and I have also seeked help from this link Fourier Transform of Derivative but I am still confused ...

 
6:43 PM
-1
Q: Why did I fail following review audit?

Bartłomiej SemańczykThe review audit is here. Below is screenshot: LOW QUALITY POSTS - I tapped EDIT. Why did I fail? It is completely well written sql query. It is not formatted, this is why I choise EDIT. Now I need to wait 7 days. It is not spam, it is quite correct answer. You may vote me down as you wish:-...

0
Q: Possible bug with displaying birthday text before adding bitrthday on Profile Page

Mike BIf you don't have your birthday set in your Stack Exchange profile, then currently the following is displayed: Seems like there is a UI issue on Firefox 42.0 where that field's description is cut-off.

 
0
Q: A question on a certain block decomposition of semi-definite matrices.

JQXLet $m,n\in\mathbb{N}$, with $m,n>1$. Suppose $K\in \mathbb{M}_{mn\times mn}(\mathbb{C})$ is positive semidefinite. We can always write $$K=\sum_{i,j=1}^m E_{i,j}\otimes K_{i,j},$$ for some collection of matrices $K_{i,j}\in \mathbb{M}_{n\times n}(\mathbb{C}) $, where $E_{i,j}\in \mathbb{M}_{m\...

0
Q: linear algebra about isomorphism

samiThe two dimensional standard vector space is not a subspace of the three dimensional standard vector space. Give three distinct subspaces of C3 that are isomorphic to C2.

0
Q: Hopf link and degree of a map

ArteomI'm considering a problem of computing the degree of a map $\varphi: S^{1} \times S^{1} \rightarrow S^{2}$ defined as $$\varphi(x, y) = \frac{\gamma_{1}(x)-\gamma_{2}(y)}{|\gamma_{1}-\gamma_{2}|}$$ $\gamma_{1}(x) = (\cos{x}, \sin{x}, 0) $ given by $x^{2}+y^{2}=1$ and $\gamma_{2}(x) = (\cos{y}+1...

 
0
Q: Where to get updated StackOverflow logo SVG version?

augNot sure if this is the correct place but I was wondering if someone knew where to get an SVG version of the updated StackOverflow logo? There is this answer but I realized it uses the old logo. I think it would be a simple color change so I attempted to do the color change but it isn't exactly ...

 
7:03 PM
0
Q: Definite integral $\int _{0}^{1}\sqrt [3] {2x^{3}-3x^{2}-x+1}dx$

fsuluovaEvaluate this integral. $\int _{0}^{1}\sqrt [3] {2x^{3}-3x^{2}-x+1}dx$

Short question. [Definite integral $\int _{0}^{1}\sqrt [3] {2x^{3}-3x^{2}-x+1}dx$](math.stackexchange.com/q/1566280)
0
Q: Problem about Foruier series and $L^p$ spaces

Alexei0709Need some help with this problems: Is there $f \in C(\mathbb{T})$ such that $\hat{f}(k) = \dfrac{1}{|k|^{1/2}}$, if $k \neq 0$? Suppose the $f_n \in L^1(\mathbb{T})$, $n = 1,2,...$ and $\| f_n - f \|_{L^1(\mathbb{T})} \xrightarrow{n \to \infty} 0$. Prove that $\hat{f_n} \xrightarrow{u} \...

0
Q: Extremum of a function

student93How to prove that $f: l_1 \to \mathbb{R}$ s.t. $$f((x_n)_{n \in N})= \sum_{n=1}^{\infty} (\frac{1}{n} x_{n}^{2}+ x_{n}^{3})$$ is a function of class $C^{\infty}$ and $f'(0)=0$ and $f''(0)(h,k)>0$ for $h,k \in l_1 \setminus \{0 \}$ but $f$ has no extremum at $0$

Consider replacing (analysis) with a more specific tag for the relevant branch of analysis. (from a bot)Normal Human 21 secs ago
0
Q: change of variables

shoestringfriesFor the problem attached, I don't understand where the 2nd double integral came from in line 4 of finding the density of $Z$. I understand up to line 3 of solving for $F_z(Z)$

Short title. Short question. change of variables
 
7:20 PM
1
Q: Difference between \mathbb and \Bbb.

user236182It seems \Bbb is obsolete compared to \mathbb (see this question on Tex.SE), but I've had no problems with \Bbb, and \Bbb is shorter than \mathbb. What are the possible dangers of using \Bbb, and should we all use \mathbb?

-1
Q: What makes geeks tick?

AndersApparently we have all been the objects of a scientific study: Many online platforms such as Yahoo! Answers and GitHub rely on users to voluntarily provide content. What motivates users to contribute content for free however is not well understood. In this paper, we use a revealed prefer...

 
0
Q: How do I answer this d=rt same direction distance problem without constant numbers in the question?

weatherman115So I found a particular question on my math homework today that I thought would be fun to toy with: Mary rides her bike the same distance that Leah walks. Mary rides her bike 10km/h faster than Leah walks. If it takes Mary 1 h[our] and Leah 3 h[ours] to travel that distance, how fast does e...

0
Q: Augmented matrixes

Linus JohanssonWe have the following matrix $$ \begin{bmatrix}1 & -2 & -1 & b_{1} \\0 & -2 & -2 & b_{1}+2 b_{2} \\0 & 0 & 0 & 3b_{1}+ \frac{7}{2}b_{2}+ b_{3} \end{bmatrix}$$ which clearly does not have any solutions when $3b_{1}+\frac{7}{2}b_{2}+b_{3} = 0$. But can we not figure out more restrictions? I...

Short title. Augmented matrixes
0
Q: Puzzle question (trick)

Jean PaulIs there a way to proof that this always works: Pick 21 different cards; Choose one card, and remember it; Place the cards on three accumulations (every accumulation is 7 cards); Choose the one with your chosen card; Pik up the three accumulations with the one with your card on the top; Do that...

Words such as question are uninformative in titles. Please edit the title so that it better describes the specifics of your question. Do not hesitate to make it longer or include a formula if needed. More tips here. (autocomment)Normal Human 22 secs ago
 
1
Q: How to tag questions regarding the open source edition of Swift

Martin ROn December 3, 2015, Swift was released as open source project on https://swift.org. Since then, several swift question were posted which refer particularly to this open source edition, and not to the Swift compiler/runtime that comes with Xcode on OS X. Some (non-representative) examples: A...

6
Q: Let's help chat rooms that want to self-moderate do so better

Jon ClementsWe have the kick-mute system in place already for RO's able to warn/temporarily remove a nuisance user from their room. "Kicking" a user gets a progressively further removal of that user's privilege to chat for a certain amount of time (in three stages). After three kicks (in various ways), the ...

 
0
Q: Compute a power series

Elimination Compute $$\sum_{n\ge 1} n^2x^n$$ Here's what I did: $$\sum_{n\ge 1} n^2x^n = x\sum_{n\ge 1} n^2x^{n-1}$$ We integrate the power-series and get: $$x\sum_{n\ge 1} nx^n = x^2\sum_{n\ge 1} nx^{n-1}$$ We integrate one more time to get: $$x^2\sum_{n\ge 1}x^n$$ In the interval $(-1,1)$ we hav...

Short title. Tagged proof-verification. Compute a power series
0
Q: I know how to solve equations of t, but I'm not sure what t' represents in this equation????

Clifford RostomilyI am investigating the Wilson Cowan neuron population model, and I can follow most of it, but I'm not sure what is meant by t' in the equation for proportion of neurons in the refractory period. Heres a link to the article: https://en.wikipedia.org/wiki/Wilson%E2%80%93Cowan_model The image below ...

0
Q: Conditional on a random variable

user297048Can someone explain to me what do we mean by Conditional on a random variable(discrete)?

0
Q: Find the supermum and infimum of a set

Shachar OrenI Got the set $\sqrt[k]{k+1}$ I have found the supermum by the Inequality of arithmetic and geometric means. And the result is 2. I dont have a way for solving the infimum . I tried to solve it by move to to some eqaution and try by Binomial theorem, but i didnt success. I know that the infimu...

Welcome to Math.SE, Shachar Oren. Questions tend to get more attention when they have a tag for a broad area of mathematics relevant to the question. Some of these tags might fit. (autocomment)Normal Human 21 secs ago
0
Q: Problem about limit points in the standard topology of $\Bbb R$

MasacrosoI have a problem about topology that is supposed I must prove without the use of any metric in the standard topology of $\Bbb R$ (metrics are not started in the book by now) just using pure topology definitions. The problem say Let $S\subset\Bbb R$ and $a\in\Bbb R$. Prove that $a\in \overlin...

Title contains problem. Tagged proof-verification. Problem about limit points in the standard topology of $\Bbb R$
0
Q: absorption laws don't understand it with venn diagrams

Valerio ZhangAbsorption Law states A∪(A∩B)=A and A∩(A∪B)=A I can't seem to picture these with venn diagrams can someone help me out ? Thanks

Questions tend to get more attention when they have a tag for a broad area of mathematics relevant to the question. Some of these tags might fit. (from a bot)Normal Human 21 secs ago
0
Q: linear algebra about automorphism

S.MutluLet ß be an automorphism of C1 such that ß(2) = 10. Compute these three conditions. 1. ß(0) 2. ß(2i) 3. ß(8 + 2i) Additionally,Can you explain automorphism with another example?

0
Q: Let z = 1 + i. Find the real and imaginary parts of z^19

BNSlugLet z = 1 + i. Find the real and imaginary parts of i.

0
Q: Help to solve $(\Delta-\lambda^2) u(\vec r)=cst$

DavidI wish to solve this guys in 3 dimensions $(\Delta-\lambda^2) u(\vec r)=-c$ where $\lambda$ and $c$ are real and $c$ is a constant. It is the screened Poisson equation and from the wikipedia page I see that the solution should be $ u(\vec r) = \int d^3r' \frac{e^{-\lambda |\vec r - \vec r'| }}...

Words such as help are uninformative in titles. Please edit the title so that it better describes the specifics of your question. Do not hesitate to make it longer or include a formula if needed. More tips here. (from a bot)Normal Human 21 secs ago
0
Q: linear algebra question

Tolga AtarFind two distinct isomorphisms between the standard nine dimensional vector space C9 and3*3 matrices M33?

Welcome to Math.SE, Tolga Atar. Words such as question do not add information to titles. Please edit the title so that it better describes the specifics of your question. Do not hesitate to make it longer or include a formula if needed. More tips here. (autocomment)Normal Human 21 secs ago
Welcome to Math.SE, amirhossein. Consider adding a tag for a broader subject area to which the question belongs. Some of these tags might fit. (from a bot)Normal Human 21 secs ago
0
Q: What happened here (Sigma Sign)?

ColdStormyhttp://i.imgur.com/wasyzxz.png?2 The red part is my main problem. I don't get why I can change just the upper border. Can somebody please explain? I tried so many ways and failed..

Short question. Question contains please. What happened here (Sigma Sign)?
0
Q: $lim_{x\to 0} \frac{e^x-1}{x^2}$

gbox$$lim_{x\to 0} \frac{e^x-1}{x^2}$$ it is an expression in form of $(\frac{0}{0})$ using l'hopital: $$lim_{x\to 0} \frac{e^x}{2x}$$ The expression in form of $(\frac{1}{0})$ so one-sided limits should be checked $$lim_{x\to 0^{-}} \frac{e^x}{2x}=-\infty$$ $$lim_{x\to 0^{+}} \frac{e^x}{2x}=\in...

A title should not be all-MathJax; having some plain text helps with search and navigation. (autocomment)Normal Human 21 secs ago
0
Q: What is the probability of getting 1or 3 in exactly 5 rolls out of when the die is rolled 6 times

galkapuThank you It would be much appreciated if I get to know the answer as soon as possible Thanks again

0
Q: Check process is a martingale

IevgeniiI have such stochastic process with which I struggle all day, finally I found 2 mistakes, however answer is still unsatisfying. $$X_t = atW_t^2 - \int_0^t(W_s^2+s)ds,$$ I need to check if it is a martingale. I simply write Ito formula for $X_t(t,W_t,S_t)$, where I denote by $S_t = \int_0^tW_s^2d...

0
Q: Need help to prove this given matrix is positive definite

user118494The matrix given to me is : $$A=$$ $$1\ \ r\ \ r\\r\ \ 1\ \ r\\r\ \ r\ \ 1.$$ Find the values of $r$ for which this is positive definite. So,I naturally try to find the determinant of the matrix $$A-xI$$ where $$I$$ is the $3\times 3$ identity matrix. The determinant is $${(1-x)}^3-2r^2(1-x)...

Words such as help do not add information to titles. Please edit the title so that it better describes the specifics of your question. Do not hesitate to make it longer or include a formula if needed. More tips here. (autocomment)Normal Human 21 secs ago
0
Q: Is this a valid proof for the triangle inequality in $\mathbb{R}^n$ (subtraction form)?

Clarinetist$\newcommand{\norm}[1]{\left\lVert#1\right\rVert}$ Let $\mathbf{x},\mathbf{y} \in \mathbb{R}^n$. I wish to show $$\norm{\mathbf{x}}-\norm{\mathbf{y}} \leq \norm{\mathbf{x}-\mathbf{y}}\text{.}$$ I have already proven $$\norm{\mathbf{x}+\mathbf{y}} \leq \norm{\mathbf{x}}+\norm{\mathbf{y}}\text{....

0
Q: A problem about $-max$ and $min$

user91360Suppose $X$ and $Y$ are convex compact subsets in $\mathbb R^n$. Let $\langle.,.\rangle$ be the standard inner product. Does the following equality $$\max_{y\in Y} [\langle y, z\rangle- \max_{x\in X}\langle y, x\rangle]=\max_{y\in Y} \min_{x\in X}\langle y, z-x\rangle $$ hold? I saw the term o...

Title contains problem. A problem about $-max$ and $min$
0
Q: Finding the Levy measure

DamianI am struggling with the derivation of the Lévy-measure of a Gamma-process $X_t$ with law $p_t(x)= \frac{\lambda^{ct}}{\Gamma(ct)}x^{ct-1}e^{-\lambda x}1_{\lbrace x>0 \rbrace }$. The paper I am reading says it is shown "easily" via the characteristic function and using the Levy-Khintchine formula...

0
Q: Find the equation of tangent line to $(x^2+y^2)^(3/2) = 2xy$ at ($1/sqrt(2)$, $1/sqrt(2)$)

Ilia LabkovskyI just can't figure out how to take the derivative of this function!

0
Q: Configurations and pressure-temperature phase diagrams

Steven UsdanskyThe questions here arise out of investigating theoretical straight-line pressure-temperature phase diagrams for suites of minerals in chemically-similar metamorphic rocks. I've tried to phrase the questions in purely mathematical terms. All points and lines lie in a single Euclidean plane. We st...

This site uses MathJax formatting of formulas. More tips here. (autocomment)Normal Human 20 secs ago
0
Q: $A_4$ is not the direct product of its sylow subgroups?

user264885This is an exercise in Hungerford. I've tried proving this as follows (forgetting for the moment that in $A_4$, $n_2=1$ and $n_3=4$): By Sylow III, $n_2=1$ or $3$. If $n_2=3$, the number of elements of order $2$ in $A_4$ is $n_2 (2-1)=3(2-1)=3.$ Then the number of elements of order not $3$ is $...

0
Q: linearly extended

S.MutluIf the map ß from polynomials of degree at most 2 P2 to C4 de ned via 0 1 1 0 ß(0)= 0 ß(x)= 0 ß(1+x^2)=0 ß(1+x+x^2)= 0 0 1 1 0 0 2 2 2 (These all matrices.But,I don't know how to ...

This site uses MathJax formatting of formulas. More tips here. (from a bot)Normal Human 20 secs ago
0
Q: Show that "the distance from $p_0$ to $S$" exists

luka5z Prove that if $S$ is non-empty compact subset of metric space $E$ and $p_0\in E$, then $\min \{d(p_0,p) : p\in S\}$ exists. Since $S$ is compact, it will be enough to proof that function $S\ni p\rightarrow d(p_0,p)\in \mathbb{R}$ is continuous. For any $x$ in $S$ and any $\epsilon>0$ ch...

0
Q: Finding an matrix for an operator.

rogieI was attempting to find a matrix for the function x*(d/dx) in the span of the set {1,x,x^2} for the the standard dot product. Could someone guide me in how to do this?

0
Q: Transition rate Markov chain

The HomeworkerAssume that we have a continuous time Markov chain $(X_t)$ on $\{0,1\}$ and $f(t):=P(X(l)=0 \text{ for all } l\in [0,t]|X(0)=0),$ then I want to show that under the assumption that $f'(0)$ exists, we have $$f'(0)=\frac{d}{dt}|_{t=0} P_t(0,0).$$ I managed to show the trivial inequality, i.e. $$...

0
Q: How to show a mapping T is Linear

Cesar AgamaSuppose T is a mapping such that T( 1,1 ) = ( 2,1 ) and T(1,-1) = (2,-1) (each set of numbers is a vector) Can the mapping T be Linear? Please note that the question is not asking if T is a Linear Transformation. Is there a way to show that T is Linear by plotting vectors and show linearity?...

Question contains please. How to show a mapping T is Linear
0
Q: Product rule in limit

gboxI have looked on Wolfram for the limit of $lim_{x \to 0} \frac{lnx}{x}$ The full answer uses one-sided limits, and the product rule stating that for $0^{-}$ is a product of $-\infty \cdot -\infty=\infty$ and for $0^{+}$ is a product of $-\infty \cdot \infty=-\infty$ does the product rule stands...

0
Q: Does almost sure continuity imply continuity of the covariance function?

XKXDoes sample paths continuity of a real-valued square integrable stochastic process imply continuity of its covariance function?

0
Q: Hypothesis Testing under uniform distribution

JennyA recent article in Healthy Life magazine claimed that the mean amount of leisure time per week for European men is 48.5 hours. The distribution of leisure time amounts is reported to be approximately Normal. You believe the figure of 48.5 is too large and decide to conduct your own test. In a ra...

Welcome to Math.SE, Jenny. Questions tend to get more attention when they have a tag for a broad area of mathematics relevant to the question. Some of these tags might fit. (from a bot)Normal Human 20 secs ago
0
Q: Equation of a Tangent Line Using Trig Derivatives

Shavana.Scan someone please help? This problem asks to find the equation of the tangent line to y= 3sin(x) at x= pi/3. I tried finding two points of the graph and finding the slope of the line by taking the derivative of 3sin(x), am I in the right direction? I think I'm doing something wrong because I ...

 
8:32 PM
0
Q: Can I speak Persian(other than English) in comments?

Hoseyn HeydariOne person from Iran write a question and there is no answer to it. If we discuss in Persian may be I can understand better his problem and answer it. Can I do it generally in comments, with Persian people?

0
Q: Function taking more than one values

QwertySuppose we want to write about a function in a question that takes more than one values . In our note books we generally write f( x ) = and then we give a big curly left bracket and then define various values the function is to take in different domains. How to write this here? I mean can someone...

 
0
Q: Division Algorithm and Cayley Tables.

J.D.1204Write out the addition and multiplication tables for $\mathbb{Z} _2$[x]/$(x^2 + 1).$

0
Q: How to determine that field with q element exists?

user3508536For instance: A Field with 13 elements exist. How to prove that this statement is true or false?

0
Q: Chessboard-Induction

gfeLLaSI am in trouble with the following excercise: We have an infinite chessboard with squares , where all squares are at first white except one initial set M(0) with n black squares,where M(0) is an initial formation of black squares.We set new formations of black square as follows: one black square ...

Short title. Chessboard-Induction
0
Q: Analiticity of a function

John RailmanDoes this definition contain redundancy: "A function $f$ is said to be analytic at $x_0$ if its Taylor series about $x_0$ exists and converges to $f(x)$ for all $x$ in some interval containing $x_0$". Is anything missing in the following definition: "A function $f$ is said to be analytic at $x_0$...

 
8:54 PM
0
Q: linear algebra about compute automorphism conditions

Jack31Let ß be an automorphism of C1 such that ß(2) = 10. Compute these three conditions. 1. ß(0) 2. ß(2i) 3. ß(8 + 2i) C1 is a vector space.

0
Q: Indefinite Integrals Using Natural log

Shavana.SHi can someone please help? I need to evaluate this indefinite integral: ((lnx)^5 dx)/(x) I know I need to use substitution, so if I let u= x but I can't figure out the antiderivative for the top portion. Thank you!

Question contains please. Indefinite Integrals Using Natural log
0
Q: Integral of $x^{-1}$ with respect to Brownian motion?

CSAWhat is the solution to the integral of the integral: $\int_0^T 1/t dB_t$? How do i go about solving this?

0
Q: What does this notation mean? $(EX)^{(k)}$

Ernad leroI am reading some notes on probability theory, and it says $$V(X) = EX^{(2)} - (EX)^{(2)}$$ Why is it using parenthesis around the powers?

0
Q: Waring's Problem and the floor function - solving a recurrence relation by hand

IntegrandFirst-time poster here. While doing some research on Waring's problem and the term $\{(3/2)^n\},$ I determined that the following recurrence relation holds for a certain sequence (here $n$ is a fixed, positive integer): $$r_{k}={n \choose k}+\frac{1}{2}r_{k-1}$$ Mathematica gave me the result $...

0
Q: For F = ℤ_11 find a basis for the subspace of F^4 consisting of solutions of 6x + 2y + 3z + 5w = 0

kbk37What is really confusing me about this problem is that the field is ℤ_11; if F = ℝ the basis could be {(-1/3,1,0,0),(-1/2,0,1,0),(-5/6,0,0,1)}. However, all three of these vectors contain values not in ℤ_11, so I'm not quite sure how to proceed.

0
Q: What is the probability that a square matrix with random entries will be indefinite?

Armen AghajanyanThe title says it all. I'm not sure on how to approach this problem. Any help will be much appreciated.

Welcome to Math.SE, kbk37. This site uses MathJax formatting of formulas. More tips here. (from a bot)Normal Human 28 secs ago
0
Q: Are $ \mathbb{A}^n (k ) = k^n $ and $ \mathbb{P}^{n} (k) = \mathrm{Proj} \ k[X_0 , \dots , X_n ]$ irreducibles?

Lina45Are $ \mathbb{A} (k ) = k^n $ and $ \mathbb{P}^{n} (k) = \mathrm{Proj} \ k[X_0 , \dots , X_n ]$ irreducibles when k is a domain ? Thanks in advance for your help.

Short question. [Are $ \mathbb{A}^n (k ) = k^n $ and $ \mathbb{P}^{n} (k) = \mathrm{Proj} \ k[X_0 , \dots , X_n ]$ irreducibles?](math.stackexchange.com/q/1566491)
 
0
Q: When asking a question, 'How To Ask / Format' panel covers footer when scrolled down

DruzionWhen asking a question there is a panel at the side stating how to ask or format a question. When you scroll down the page, the panel overlaps the footer (see pictures). This looks quite ugly, especially on physics.stackexchange.com, since the panel has no background color. I am assuming this is...

 
0
Q: 2D line passes through two points, what is the perpendicular distance from point P?

GhostSuppose a 2D line passes through two points P0(10, 15) and P1(200, 20) What is the perpendicular distance from point P(-500, 48) to the line? Following this formula this is as far as I got before I got lost. dx = 10-200 dy = 15-20 dist = sqrt(-190*-190 + -5*-5) dx / 190 (dist) dy / 190 (dist...

This site uses MathJax formatting of formulas. More tips here. (from a bot)Normal Human 21 secs ago
0
Q: Sum of binomial coefficients multiplied by k^2

ThomasEvaluate $\displaystyle\sum\limits_{i=1}^{12} {n\choose{k}}k^2$ The answer is 159744.

Title ends with a digit. Short question. Sum of binomial coefficients multiplied by k^2
0
Q: How can I write the linear system of equations given by the Oseen-iteration in the form $Ax=b$?

0xbadf00dLet $d\le 3$ and $\Omega\subseteq\mathbb R^d$ be a bounded domain. I'm considering an incompressible Newtonian fluid with uniform density $\rho_0$ and viscosity $\nu$. In this case, the stationary Navier-Stokes equations are $$\left\{\begin{matrix}(u\cdot\nabla)u&=&\displaystyle\nu\Delta u-\frac ...

Tagged pde, differential-equations. Tagged differential-equations but mentions "partial". How can I write the linear system of equations given by the Oseen-iteration in the form $Ax=b$?
0
Q: How to prove Dvoretzky–Kiefer–Wolfowitz inequality

lukeHow can we verify Dvoretzky–Kiefer–Wolfowitz inequality for a uniform Distribution?

0
Q: Few group theory questions

PersonaAI am trying to solve the following; First, given G is a group and H a subgroup of G, what can we say about the relation $a \cong b$ if $b^{-1}a \in H$ I can show that it is reflexive as the identity is always in subgroup. if $a \cong b$ then $b^{-1}a \in H$ and so $(b^{-1}a)^{-1}=a^{-1}b$ $\i...

0
Q: How can this recurrence relation be solved?

user249117We are provided with a recurrence relation as follows:- F(n,k)=F(n-k,k-1)+F(n-k,k) ; F(n,0)=0 ; F(1,1)=1 I need help to solve this one.

0
Q: Indefinite Integral Using Substitution

Shavana.Scan someone please help? I need to evaluate this indefinite integral: (x)dx /(sqrt{x^2+ 2}) I tried using substitution for letting u = x, but I can't get past finding the antiderivative after that. Thank you!

Question contains please. Indefinite Integral Using Substitution
0
Q: Unprovable behavior of a turing machine

user292477The wikipedia-article for the P-NP problem [1] says there are three possible answers to the P-NP-problem: $P=NP$ $P\neq NP$ $P=NP$ is independent of ZFC The third possible solution seems to be very interesting. Assuming it is true, there could still exist a turing machine which solve e.g. $SA...

Consider adding a tag for a broader subject area to which the question belongs. Some of these tags might fit. (from a bot)Normal Human 21 secs ago
0
Q: Compactness of the Operator Tx(t)=tx(t)

user105570Is the operator $A:C[0,1]\to C[0,1]$ defined by $Ax(t)=tx(t)$ is compact? And what if we change the underlying space?

0
Q: Spectral Theorem for kompact Operators

Johnson DonI think about the spectral theorem for compact operators on a Banach Space. And I come to a question: Can the Theorem be generalized to any Normed Space or a bigger subclass of TVS

 
9:35 PM
0
Q: Helping a user - Question clean up

user3788685I come looking for some guidance on how to best help a user in a couple of ways. They have come asking a question, which is a follow on from this question on Superuser about a specific problem. But in answering there question I've actually found out they need some general help with an over-all s...

 
0
Q: Compound Interest With Regular Deposits

Julian73I know of: $a = p(1+\frac rn)^{nt} , $ but how would this be modified to include a regular deposit at a given interval. Thanks for your help, Julian

0
Q: How do i evaluate this integral for any integer $n>1$ $\int \limits _0^ {\infty} ln^n\frac{|1-x|}{|1+x|}dx$?

Salmahamizi HamiziHow do i evaluate this integral for any integer $n>1$ $$\int \limits _0^ {\infty} ln^n\frac{|1-x|}{|1+x|}dx$$ Note: for $n=1$ just we use integral by part and the integral will be divergent .My problem what about $n >1$ Thank you for any kind of help

 
0
Q: Time Dilation Special Relativity

samsoniteAssume that 437 days is a reasonable limit for how long a human can endure constant-velocity space travel. Proxima Centauri, the star closest to our Sun, is 4.24 light years away from Earth. If you wanted to fly to Proxima Centauri within the 437-day limit in a rocket of mass 2.00×10^6 kg , how m...

 
0
Q: Computing curvature of vector fucntion

courageuxI have the following question given r(t) = e^(t) cos(t) i + e^(t) sin(t) j, where 0 < t < infinity reparametrize the curve by arc length and compute its curvature. I know how to reparametrise it but can't seem to use the reparametrisation to computer curvature. Help

This site uses MathJax formatting of formulas. More tips here. (from a bot)Normal Human 21 secs ago
0
Q: How to prove $r^3+1 = (r+1)(r^2-r+1)$

diogenes$r^3+1 = (r+1)(r^2-r+1)$ I know we can simply multiply equations in the right-hand side then we get $r^3+1$. However, is there any way to construct right-hand side without knowing it?

0
Q: A problem with the scalar product of a free vectorial space

Adrian BerteanuLet V the space of free vectors from the geometric space. Prove that: =|AB|*|AC|*cos(<(AB,AC)) is a scalar product, where AB and AC are vectors and <(AB,AC) is the angle between AB and AC counted in the trigonometrical sense. I tried choosing a base in V,so I can get the biliniar form fr...

This site uses MathJax formatting of formulas. More tips here. (from a bot)Normal Human 21 secs ago
0
Q: Why $|a_n|^{1/n}<1/r$?

user42912I'm reading Conway's complex variable book and I didn't understand this proof on page 31: I didn't understand why $\frac{1}{r}>\frac{1}{R}$ implies there is an integer $N$ such that $|a_n|^{1/n}<1/r$ for all $n\ge N$. Any help is welcome.

0
Q: What "linguistic and philosophical problems" might be inherent in trigonometry?

natevwIn "A Mathematician’s Lament", Paul Lockhart derides the "status quo" of math education, claiming that "mathematics is an art form done by human beings for pleasure" but instead is taught "devoid of creative expression of any kind". His writing is provocative, and I'm sure his accusations and sug...

0
Q: Quadrics on Severi varieties

Srinivasa GranujanI am trying to understand proposition 2.1 of chapter IV of Zak's 'Tangents and Secants of Algebraic Varieties'. Let $$ S^{0}_{X}=\{(x,y,z):(x,y)\in X^{2}-\Delta_{X}, z\in \langle x,y\rangle \}, $$ $$ S_{X}=\overline{S^{0}_{X}}\subseteq X\times X\times \mathbb{P}^{N}. $$ We denote $$ p_{1}:S_{X}\...

0
Q: regression for weak records

rafalpwLet $\left( X_n \right)_{n \ge 1}$ be a sequence of iid random variables with discrete distribution $p_k=P(X=k)>0$ for $k=0,1,2,...,N$, where $N<\infty$ and $\sum_{k=0}^Np_k=1$. We define $T_1=1$ and $T_n=\min \{ k \in \mathbb{N} : k>T_{n-1} \wedge X_k \ge X_{T_{n-1}} \}$ for $n=2,3,...$ and $W_...

0
Q: Applying Rouche's Theorem

complexLostFor a complex polynomial F(z) = z^8 + 5z^7 - z^4 + 2, how many roots does it have in the disk |z| < 1? I want to apply Rouche's Theorem. So let f(z) = 5z^7. Then |F(z) - f(z)| = |z^8 - z^4 + 2| <= |z^8| + |-z^4| + |2|. At z=1, 1 + 1 + 2 = 4 < 5 = |f(z)| which implies F(z) has 7 roots in |z|<1. D...

This site uses MathJax formatting of formulas. More tips here. (from a bot)Normal Human 21 secs ago
 
10:37 PM
0
Q: Arithmetic Sequence with conditions

ChrisThe question is: Let {$a_j$} be an arithmetic sequence with the following conditions: $S_16$ = 376 and $a_16$ = 46. Find $a_1$

 
10:54 PM
0
Q: Picards theorem problem rectangle validity of solutions depending on a and b

Arcane1729 Could I have help with part biii) please? I solved b ii) by getting an equation for $f(x,y)$ by substituting from the differential equation and getting bounds for all the terms like $\frac{1}{x}$ , $exp(\frac{1}{x})$ using the bounds on $x$ and $y$ given but obviously couldn't get a specific $...

0
Q: Evaluating a line integral along a curve...?

teaberryjI am supposed to evaluate F(x,y)=(4x^3y^2-2xy^3)i + (2x^4y-3x^2y^2+4y^3)j along the curve r(t)=(t+sin(tpi))i+(2t+cos(tpi)j, 0<=t<=1. I could put try to do the \int f(r,t) r'(t) as t goes from 0 to 1, but that seems really messy. Is there a simpler way?

Welcome to Math.SE, teaberryj. This site uses MathJax formatting of formulas. Consider adding a tag for a broader subject area to which the question belongs. Some of these tags might fit. More tips here. (from a bot)Normal Human 21 secs ago
0
Q: please help! my mom thinks finite math and organic chemistry is the same.

cassie h.Well I'm in organic chemistry and am trying to explain to her that it's not the same thing at all. Please help me explain it to her.

Title contains please, help. Short question. Question contains please. please help! my mom thinks finite math and organic chemistry is the same.
 
1
Q: Questions regarding running Apache on Windows

EEAAWe get several questions a day from people trying to run Apache on Windows platforms. Excluding WAMP questions (which are already off-topic), should we allow questions about running Apache on Windows? Apache is clearly designed to run on *nix operating systems. While it's possible to run it on W...

 
0
Q: tangent vector to curve of intersection of 2 cylinders

courageuxFind the vector tangent to the curve of intersection: x^2 + y^2 =8 and y^2 +z =8 at the point (2,-2,2)

This site uses MathJax formatting of formulas. More tips here. (from a bot)Normal Human 21 secs ago
0
Q: Help Solving Fraction Math Question

j76goatboyIm stumped by this question on a practice ACT math test. If $\frac 1x + \frac 1y = \frac 1z$ then $z =$? The correct answer is $\frac {xy}{x + y}$ How do you arrive at this answer? I don't know how to even begin with this problem.

Words such as help, question do not add information to titles. Please edit the title so that it better describes the specifics of your question. Do not hesitate to make it longer or include a formula if needed. More tips here. (from a bot)Normal Human 20 secs ago
0
Q: Simplifying complex numbers

QuestionnaireI wanted to ask whether my thoughts are right. After computing out the angle of π/4 and the r I got this: z_{1} = 0,5^{100}\exp(\pi*i) Did I make any faults? Could it be simplified any further?

This site uses MathJax formatting of formulas. More tips here. (autocomment)Normal Human 21 secs ago
0
Q: Limit existence and explanation thereof

JonnyMy problem revolves around the function: $$ f(x) = \frac{\sin(\tan x) - \tan(\sin x)} {\arcsin(\arctan x) - \arctan(\arcsin x)} $$ The limit of f(x) as x -> 0 = 1. However, whilst approaching 0, there is a lot of sporadic oscillations (mainly between -0.005 and 0.005). I've been asked 'Since ...

0
Q: Help in a calculs

Z. AlfataLet: $$x= \frac{1}{\cosh^{2} (t)},$$ I want to express $\frac{d^n}{dx^n}$ in term of $\frac{d}{d t}$. Thanks in advance

Words such as help do not add information to titles. Please edit the title so that it better describes the specifics of your question. Do not hesitate to make it longer or include a formula if needed. More tips here. (from a bot)Normal Human 21 secs ago
0
Q: Simple direction derivative

d0rmLifeI'm having some trouble solving this directional derivative problem. $$ f(x,y) = sin(2x + 5y), P(-15, 6), \mathbf{u} = {1 \over 2} (\sqrt{3}\mathbf{i-j}) $$ I know the theorem for the bivariate directional derivative: $$D_u f(x, y) = f_x(x, y)a + f_y(x, y)b $$ But that is with the unit vector...

0
Q: Fake proof that there don't exist complicated numbers

Martín Forsberg CondeSo there's this false proof going around that I can't seem to find now that says that complicated numbers don't exist. So let me explain what it's about (I've added some technical details of my own, but the idea is the same). Definition A number $a \in \mathbb{N}$ is said to be complicated if it...

Tag (fake-proofs) should not be the only tag a question has. Please add a tag for a subject area to which the question belongs. (autocomment)Normal Human 21 secs ago
0
Q: Let G be a graph. Let k be the minimal degree of G. Show that G contains a cycle of length k + 1.

user297125Let G be a graph. Let k be the minimal degree of G. Show that G contains a cycle of length k + 1.

0
Q: Solving for y(x) explicitly.

misheekohSo, I have y as an implicit function $e^{2x}= ln(\frac{2(y-1)}{y+1}$) Any hints to how I can get y explicitly? Any hints / help?

Short title. Short question. Solving for y(x) explicitly.
0
Q: KKT Conditions for LP

clocktowerHow may I state the KKT conditions for minimize $c^{T}x$ subject to $Ax \leq b$, $x$ unrestricted?

Short title. Short question. KKT Conditions for LP
 
11:38 PM
0
Q: Unable to access "90d" tab on "question-close-stats" page

Paranoid PandaWhen I try to go to this page I get this error:

 
0
Q: Solving Polar Equations

user242559How do you solve $\sin\theta$+$\cos\theta$=$1.2?$On the interval{$0$,$2Pi$}. Answer in the nearest radian. I've got... cos(x)=sqrt($1-sin^2x$) sin(x)-sqrt($1-sin^2x$)=1.2 (sin(x)+1.2)=(sqrt($1-sin^2x$)^$2$ $sin^2x$+2.4sin(x)+(1.44-1)=.44 $sin^2x$+2.4sin(x)-.44=0 $2$$sin^2x$+2.4sin(x)-.44=0...

0
Q: Matlab Trapezoid Rule Code

blarghOkay so I'm doing numerical integration using the trapezoid rule. I wrote the m file as such: function y = trap(f,a,b,N) h= (b-a)/N; x=0; for i=1: N-1 x=(a+i*h);y=y+eval(f); end y=2*y; x=a; y=y+eval(f); x=b; y=y+eval(f); y= (h/2)*y; It says that the value assigned to x might be unused and ...

Welcome to Math.SE, blargh. This site uses MathJax formatting of formulas. More tips here. (from a bot)Normal Human 21 secs ago
 
00:00 - 16:0016:00 - 00:00

« first day (36 days earlier)      last day (538 days later) »